High School Test Prep

AP World History Long Essay Question Example 1

Have you written out your own response to the first of our AP World History Long Essay Questions ? Below you can review a sample answer and evaluate what earns this AP World History LEQ example a perfect score.

Evaluate the impact of the trans-Saharan trade routes on the exchange of goods, cultures, and ideas in Africa and the Islamic world during the period c. 1200–1750.  Analyze the role of trade networks in shaping societies and economies during this era.

Sample Answer:

Thesis/Claim (1 point):  The trans-Saharan trade routes played a pivotal role in facilitating the exchange of goods, cultures, and ideas between regions in Africa and the Islamic world during the period c. 1200–1750, leading to significant economic, cultural, and technological advancements. Contextualization (1 point):  Before the emergence of the trans-Saharan trade routes, both Africa and the Islamic world experienced dynamic historical developments. In Africa, the Kingdom of Ghana flourished as a major trading empire, while in the Islamic world, the spread of Islam led to increased cultural diffusion and intellectual growth. These developments set the stage for the establishment of the trans-Saharan trade networks. Evidence (2 points):  The trans-Saharan trade routes facilitated extensive exchanges between regions. One specific example of this exchange is the trade of gold from West Africa for salt and textiles from North Africa. The availability of gold in West Africa, particularly in the Mali Empire, spurred demand for North African goods. This trade not only enriched both regions economically but also led to cultural exchanges. Furthermore, the introduction of new crops such as sorghum and millet to West Africa from the Islamic world improved agricultural practices, leading to increased food production and population growth. Another significant development is the spread of Islam along these trade routes. As merchants and traders traveled across the Sahara, they also brought Islamic culture and religion with them. This resulted in the conversion of some African societies to Islam, creating a cultural and religious connection between regions. Additionally, the construction of notable centers of learning such as Timbuktu in Mali became hubs for intellectual exchange, where scholars from Africa and the Islamic world gathered to share knowledge. Analysis and Reasoning (2 points):  The trans-Saharan trade routes not only connected regions economically but also fostered cultural and intellectual exchanges. This is evident in the adoption of Islamic practices and the establishment of centers of learning in West Africa. Furthermore, the trade in goods such as gold and salt played a vital role in the economic development of both regions. To achieve a more complex understanding, it’s essential to consider that while the trans-Saharan trade routes brought about positive exchanges, they also had challenges, such as the harsh desert environment and the impact on local communities. Nevertheless, the overall impact was largely beneficial, contributing to the growth and interconnectedness of African and Islamic societies. In conclusion, the trans-Saharan trade routes during the period c. 1200–1750 were instrumental in promoting economic, cultural, and intellectual exchanges between regions in Africa and the Islamic world. This interconnectedness resulted in mutual enrichment and the spread of Islamic culture and knowledge.

Total Points: 6

This essay provides a historically defensible thesis, relevant contextualization, specific evidence, and a well-structured analysis, earning a perfect score according to the AP World LEQ Rubric.

When you are finished with LEQ question 1, proceed to the next sample answers to review other strong essays or return to the main menu with the buttons below.

PrepScholar

Choose Your Test

  • Search Blogs By Category
  • College Admissions
  • AP and IB Exams
  • GPA and Coursework

The Complete Guide to the AP World History Exam

Advanced Placement (AP)

feature_apworldhistoryexam.jpg

Before you start studying for the AP World History exam, you should get the inside scoop on its format and content. The types of questions you'll see might differ from your expectations. It's especially smart to practice writing essay outlines based on past questions before you're faced with fresh prompts on test day.

In this article, we'll go through the structure, content, and question types on the AP World History exam and provide some helpful tips for acing it !

How Is the AP World History Exam Structured?

The AP World History exam is three hours and 15 minutes and consists of two sections, each of which contains a Part A and a Part B. Here's a basic overview of these sections before we get into the nitty-gritty below:

  • Section 1, Part A: Multiple Choice
  • Section 1, Part B: Short Answer
  • Section 2, Part A: Document-Based Question (DBQ)
  • Section 2, Part B: Long Essay

It should be noted that the AP World History exam has undergone some big changes for the 2019-20 school year . Instead of covering thousands of years of human history and development, now it will cover only the years 1200 to the present ; as a result, the exam has been renamed AP World History: Modern (an AP World History: Ancient course and exam are in the works).

Section 1 Format

Section 1 on the AP World History exam lasts a total of 95 minutes and consists of two parts:

  • Part A: Multiple Choice
  • Part B: Short Answer

This chart shows what you can expect for each part of Section 1 on the World History exam:

Part A Multiple Choice 55 mins 55 40%
Part B Short Answer 40 mins 3 (for third, choose 1 of 2 prompts) 20%

Part A lasts 55 minutes and counts for 40% of your total AP World History score. Each question on this part comes with four possible answer choices (labeled A-D). Since there are no penalties for wrong answers, it's in your best interest to fill in an answer for every question.

Most multiple-choice questions come in sets of three to four questions and require you to analyze primary and secondary sources as well as data in the form of graphs, charts, maps, etc.

Part B lasts 40 minutes and counts for 20% of your final AP score. For this section, you must write three short answers (you'll get four prompts in total, but you choose one of two prompts to write on for your third essay). You'll have different sources, or stimuli, for each short-answer prompt:

  • Short Answer 1: Includes one secondary source
  • Short Answer 2: Includes one primary source
  • Short Answer 3/4 (choose one prompt): No stimulus for either option; prompt 3 focuses on the years 1200-1750, while prompt 4 focuses on the years 1750-2001

Section 2 Format

Section 2 on the AP World History test lasts 100 minutes and, like Section 1, consists of two parts:

  • Part A: Document-Based Question (DBQ)
  • Part B: Long Essay

Here's a brief overview of the format of Section 2 of the World History exam:

Part A Document-Based Question 60 mins (including a 15-min reading period) 1 25%
Part B Long Essay 40 mins 1 (choose 1 of 3 prompts) 15%

Part A in Section 2 lasts one hour and counts for 25% of your total score. For the Document-Based Question, or DBQ, you'll get seven documents offering different viewpoints on a certain historical development. You must write an essay with an argument supported by this historical evidence.

Part B, which is the Long Essay, lasts for just 40 minutes and counts for 15% of your AP score. This part of the test requires you to write a full-fledged essay in response to one of three prompts (you choose which one you want to write on). Unlike the DBQ, you're not given any direct historical evidence to use in your essay; you must come up with it yourself to support your argument.

Here are the three types of prompts you can choose from for the Long Essay:

  • Prompt 1: Focuses on the years 1200-1750
  • Prompt 2: Focuses on the years 1450-1900
  • Prompt 3: Focuses on the years 1750-2001

body_stonehenge

What Kind of Content Is Covered in AP World History?

Content on the AP World History exam is divided into six overarching themes and nine distinct units . Knowing these categorizations can help you get a better sense of what kinds of historical trends you will be asked to examine (this is especially helpful when writing free-response essays!). The units are roughly divided up into overlapping periods of time.

Below, we introduce the current themes and units, as described in the 2019-20 AP World History: Modern Course and Exam Description .

The 6 Themes in AP World History

Let's start by looking closely at the six major themes covered on the AP World History exam.

Theme 1: Humans and the Environment

The environment shapes human societies, and as populations grow and change, these populations in turn shape their environments.

Key points:

  • Demography and disease
  • Patterns of settlement

Theme 2: Cultural Developments and Interactions

The development of ideas, beliefs, and religions illustrates how groups in society view themselves, and the interactions of societies and their beliefs often have political, social, and cultural implications.

  • Religions and cultures
  • Belief systems, philosophies, and ideologies
  • Science and technology
  • The arts and architecture

Theme 3: Governance

A variety of internal and external factors contribute to state formation, expansion, and decline. Governments maintain order through a variety of administrative institutions, policies, and procedures, and governments obtain, retain, and exercise power in different ways and for different purposes.

  • Political structures and forms of governance
  • Nations and nationalism
  • Revolts and revolutions
  • Regional, transregional, and global structures and organizations

Theme 4: Economic Systems

As societies develop, they affect and are affected by the ways that they produce, exchange, and consume goods and services.

  • Agricultural and pastoral production
  • Trade and commerce
  • Labor systems
  • Industrialization
  • Capitalism and socialism

Theme 5: Social Interactions and Organization

The process by which societies group their members and the norms that govern the interactions between these groups and between individuals influence political, economic, and cultural institutions and organization.

  • Gender roles and relations
  • Family and kinship
  • Racial and ethnic constructions
  • Social and economic classes
  • Slavery and abolition

Theme 6: Technology and Innovation

Human adaptation and innovation have resulted in increased efficiency, comfort, and security, and technological advances have shaped human development and interactions with both intended and unintended consequences.

  • Intellectual innovation
  • Transportation technologies and trade
  • Modes of production and machinery
  • Communication

body_hands_world_map

The 9 Units in AP World History

Here, we'll go over the nine units of the AP World History course and exam. But before we describe each one in depth, here's a quick overview of how these units are tested:

Unit 1: The Global Tapestry 1200-1450 8-10%
Unit 2: Networks of Exchange 8-10%
Unit 3: Land-Based Empires 1450-1750 12-15%
Unit 4: Transoceanic Interconnections 12-15%
Unit 5: Revolutions 1750-1900 12-15%
Unit 6: Consequences of Industrialization 12-15%
Unit 7: Global Conflict 1900-present 8-10%
Unit 8: Cold War and Decolonization 8-10%
Unit 9: Globalization 8-10%

Source: AP World History Course and Exam Description, 2019-20

Unit 1: The Global Tapestry (1200-1450)

  • Developments in East Asia from c. 1200 to c. 1450
  • Developments in Dar al-Islam from c. 1200 to c. 1450
  • Developments in South and Southeast Asia from c. 1200 to c. 1450
  • State Building in the Americas
  • State Building in Africa
  • Developments in Europe from c. 1200 to c. 1450
  • Comparison in the period from c. 1200 to c. 1450

Unit 2: Networks of Exchange (1200-1450)

  • The Silk Roads
  • The Mongol Empire and the making of the modern world
  • Exchange in the Indian Ocean
  • Trans-Saharan trade routes
  • Cultural consequences of connectivity
  • Environmental consequences of connectivity
  • Comparison of economic exchange

Unit 3: Land-Based Empires (1450-1750)

  • Empires expand
  • Empires: administration
  • Empires: belief systems
  • Comparison in land-based empires

Unit 4: Transoceanic Interconnections (1450-1750)

  • Technological innovations from 1450 to 1750
  • Exploration: causes and events from 1450 to 1750
  • Columbian exchange
  • Maritime empires established
  • Maritime empires maintained and developed
  • Internal and external challenges to state power from 1450 to 1750
  • Changing social hierarchies from 1450 to 1750
  • Continuity and change from 1450 to 1750

Unit 5: Revolutions (1750-1900)

  • The Enlightenment
  • Nationalism and revolutions in the period from 1750 to 1900
  • Industrial Revolution begins
  • Industrialization spreads in the period from 1750 to 1900
  • Technology of the Industrial Age
  • Industrialization: government's role from 1750 to 1900
  • Economic developments and innovations in the Industrial Age
  • Reactions to the industrial economy from 1750 to 1900
  • Society and the Industrial Age
  • Continuity and change in the Industrial Age

Unit 6: Consequences of Industrialization (1750-1900)

  • Rationales for imperialism from 1750 to 1900
  • State expansion from 1750 to 1900
  • Indigenous responses to state expansion from 1750 to 1900
  • Global economic development from 1750 to 1900
  • Economic imperialism from 1750 to 1900
  • Causes of migration in an interconnected world
  • Effects of migration
  • Causation in the Imperial Age

Unit 7: Global Conflict (1900-Present)

  • Shifting power after 1900
  • Causes of World War I
  • Conducting World War I
  • The economy in the interwar period
  • Unresolved tensions after World War I
  • Causes of World War II
  • Conducting World War II
  • Mass atrocities after 1900
  • Causation in global conflict

Unit 8: Cold War and Decolonization (1900-Present)

  • Setting the stage for the Cold War and decolonization
  • The Cold War
  • Effects of the Cold War
  • Spread of communism after 1900
  • Decolonization after 1900
  • Newly independent states
  • Global resistance to established order after 1900
  • End of the Cold War
  • Causation in the age of the Cold War and decolonization

Unit 9: Globalization (1900-Present)

  • Advances in technology and exchange after 1900
  • Technological advances and limitations after 1900: disease
  • Technological advances: debates about the environment after 1900
  • Economics in the Global Age
  • Calls for reform and responses after 1900
  • Globalized culture after 1900
  • Resistance to globalization after 1900
  • Institutions developing in a globalized world
  • Continuity and change in a globalized world

body_snowball.jpg

Sample AP World History Test Questions

Let's go through examples of each of the four types of questions you'll see on the AP World History exam. All sample questions come from the 2019-20 World History Course and Exam Description .

Multiple-Choice Question Example

body_ap_world_history_mc_question_sample

Most multiple-choice questions come in sets of three to four questions that ask you to respond to a particular source, or stimulus, such as a primary source, a secondary source, or data in the form of a map, chart, or table.

In this sample question, you're being asked to read and interpret two separate passages . You must have background knowledge of economic trends in the late 20th century to be able to select the correct answer here (which is answer choice C ).

Short-Answer Question Example

body_ap_world_history_short_answer_sample_question

This short-answer question is accompanied by a secondary source. In each short-answer question on the test, each part (A-C) should only require a one- to two-sentence answer . You'll then get 1 point per correct response (so the max you can earn on one short-answer question is 3 points).

Here's how you could earn full credit for this question, per the official scoring guidelines .

(A) Sample Answers

  • Hakuseki's argument was influenced by Confucianism.
  • Hakuseki's argument that sovereign is Heaven to the subjects and the father is Heaven to the child was influenced by Confucian beliefs.
  • Hakuseki's argument that only the emperor is supposed to serve the Lord of Heaven reflects the beliefs of Confucianism.

(B) Sample Answers

  • One important difference is that most Christian missionaries and Muslim Sufis traveled across the world and spread their religion without being banned by other governments.
  • One important difference between the circumstances of the religious encounter in eighteenth century Japan and other religious encounters in the period 1450–1750 is that religious interactions in this period more frequently led to the development of syncretic belief systems such as Vodou or Santería than the outright banning of the preaching of a religion.
  • One important difference between the Tokugawa shogunate banning the preaching of Christianity and most other religious interactions in the period 1450–1750 is that some governments, such as the Mughal Empire under Akbar, encouraged religious tolerance and interaction.

(C) Sample Answers

  • The Mughal emperors of India and the African kings of Kongo attempted to restrict European merchants to certain towns and trading posts.
  • The Ming and Qing emperors of China confined the Portuguese merchants to Macao and placed legal restrictions on converting to Christianity.
  • Although the Safavid Empire allowed European merchants to settle in some cities and even serve as advisors at court, preaching Christianity was strictly forbidden.

body_investigate_man_magnifying_glass_funny

Document-Based Question Example

body_ap_world_history_dbq_sample

You'll get seven documents with your DBQ (not shown in the sample above), and you must use at least six of these as evidence in your response. The DBQ is worth up to 7 points .

Here's what you'd need to do to earn full credit for this sample DBQ, per the scoring guidelines .

Thesis/Claim
(0-1 points)
The response must provide a historically defensible thesis or claim that establishes a position on the extent to which the experience of the First World War changed relationships between Europeans and colonized peoples; the thesis or claim must either provide some indication of the reason for making that claim OR by establishing categories of the argument
Contextualization
(0-1 points)
Must accurately describe a context relevant to the First World War or interactions between Europeans and colonized peoples before or during the First World War (nineteenth and early twentieth centuries)
Evidence
(0-3 points)

Support an argument in response to the prompt by accurately using the content of at least six documents; the six documents do not have to be used in support of a single argument, but they can be used across subarguments or to address counterarguments

Must use at least one specific piece of historical evidence relevant to an argument about the extent to which there were changes in relationships between European and colonized people that resulted from the First World War

Analysis and Reasoning
(0-2 points)
Must explain how or why—rather than simply identifying—the document's point of view, purpose, historical situation, or audience is relevant to an argument that addresses the prompt for each of the three documents sourced Must demonstrate a complex understanding, such as by explaining nuance of an issue by analyzing multiple variables, or by explaining relevant and insightful connections within and across periods

Looking for help studying for your AP exam? Our one-on-one online AP tutoring services can help you prepare for your AP exams. Get matched with a top tutor who got a high score on the exam you're studying for!

Long Essay Question Example

body_ap_world_history_long_essay_sample_question

For the Long Essay, you'll get three possible prompts to choose from. This question is an example of an Option 2 prompt with a focus on the years 1450-1900 —in this case, the 19th century. You can earn up to 6 points for your essay.

Here's what you'd need to do to earn full credit for this sample, per the official scoring guidelines :

Thesis/Claim
(0-1 points)
The response must provide a historically defensible thesis or claim about the extent to which reform movements succeeded in bringing about political or social change in industrial society in the nineteenth century. The thesis or claim must either provide some indication of the reasoning for making that claim OR by establishing analytic categories of the argument

Contextualization
(0-1 points)

Must accurately describe a context relevant to reform movements or industrial society in the nineteenth century
Evidence
(0-2 points)
Must use at least two specific historical
evidence examples to support an argument regarding how reform movements brought about political or social change in industrial society in the nineteenth century

Analysis and Reasoning
(0-2 points)

Must demonstrate a complex understanding, such as by explaining nuance of an issue by analyzing multiple variables, or by explaining relevant and insightful connections within and across periods N/A

body_calculator_clip_art

How Is the AP World History Exam Scored?

Now that you've seen what questions look like on the AP World History test, let's quickly go over exactly how the exam is scored. Like all other AP tests, you will get a final scaled score for AP World History on a scale of 1-5 . This is a cumulative score that combines your raw scores from each of the four parts of the test (Multiple Choice, Short Answer, DBQ, and Long Essay).

Keep reading to see how each part of the AP World History test is scored.

Multiple Choice

Raw scoring for the multiple-choice section is simple: you earn 1 point for each multiple-choice question you answer correctly . Since there are 55 questions, the max number of points you can earn here is 55. Remember that this part counts for 40% of your total score .

No points are taken off for incorrect answers, so be sure to fill in every answer bubble!

Short Answer

Each short-answer question is worth 3 points—one for each task (labeled A-C) you must complete. Because there are three short-answer questions, this means you can earn a total of 9 raw points for all your responses. The short-answer portion counts for 20% of your final AP score .

Document-Based Question

The DBQ is worth 25% of your final score and is graded on a 7-point rubric , as shown below:

Thesis/Claim
(0-1 points)
Responds to the prompt with a historically defensible thesis/claim that establishes a line of reasoning
Contextualization
(0-1 points)
Describes a broader historical context relevant to the prompt
Evidence
(0-3 points)
Supports an argument in response to the prompt using at least six documents Uses at least one additional piece of the specific historical evidence (beyond that
found in the documents) relevant to an argument about the prompt
Analysis and Reasoning
(0-2 points)
For at least three documents, explains how or why the document's point of view, purpose, historical situation, and/or audience is relevant to an argument Demonstrates a complex understanding of the historical development that is the focus of the prompt, using evidence to corroborate, qualify, or modify an argument that addresses the question

The Long Essay is worth just 15% of your overall score and is graded on a 6-point rubric :

Thesis/Claim
(0-1 points)

Responds to the prompt with a historically defensible thesis/claim that establishes a line of reasoning

Contextualization
(0-1 points)

Describes a broader historical context relevant to the prompt

Evidence
(0-2 points)

Supports an argument in response to the prompt using specific and relevant examples of evidence

Analysis and Reasoning
(0-2 points)

Demonstrates a complex understanding of the historical development that is the focus of the prompt, using evidence to corroborate, qualify, or modify an argument that addresses the question

The Best Way to Prep for the AP World History Exam: 3 Tips

Here are a few of the most important prep tips for AP World History. If you want more advice, take a look at our article on the best study strategies for this exam .

Tip 1: Make Connections to Themes (and Memorize Examples)

World History is a course that covers a ton of information, so much so that it can be hard to think of specific examples that relate to your arguments in essay questions. You should be able to elaborate on one or two concrete events from each period that relate to each theme of the course.

As long as you can preserve this bank of information in your mind, you'll be able to support your answers to any essay questions the test throws at you.

Tip 2: Use Outside Information Selectively

Providing specific historical examples in your essay lets you show your mastery of the material, but you need to be cautious. This test is less about how much you know and more about how well you understand the connections and underlying themes that connect historical facts.

Each fact you mention must have a specific purpose and should tie directly into what the question is asking and what you've stated in your argument.

Tip 3: Learn to Read Multiple-Choice Questions Carefully

You can get into some trouble if you don't understand exactly what the multiple-choice questions are asking on this exam. You'll only find the correct answer if you stick to the specifics of the question. Otherwise, you could get tripped up by choices that are accurate statements about history but inaccurate answers to the question being asked.

Practice your skills in selecting answers that directly pertain to the evidence presented in the question.

body_sticktothepoint.jpg

Summary: What to Know About the AP World History Exam

The AP World History exam is a three-hour and 15-minute test that consists of 55 multiple-choice questions, three short answers, one DBQ, and one essay.

Questions address six major historical themes and nine units, with periods stretching back to the year 1200 CE. While this is undoubtedly a lot of information to study, it's important to realize that long-term trends are more important than small details.

You can do extremely well on the World History test, as long as you master the major events of each period and understand their essential causes and effects .

What's Next?

Looking for some practice materials for the World History exam? Then check out this detailed guide to all the AP World History practice tests available online .

It's a smart idea to practice your writing skills on DBQs before any AP history test. Learn about the best places to find DBQ examples and how you can write an excellent response .

Which AP classes should you take in high school besides World History? Our expert guide will help you decide based on your goals, academic interests, and schedule.

Thinking ahead to college applications?   If you’re a freshman, sophomore, or junior worried about college admissions, our world-class admissions counselors can help. We know exactly what kinds of students colleges want to admit and can make sure your profile shines.   PrepScholar Admissions combines world-class admissions counselors with our data-driven, proprietary admissions strategies. Start your mentoring package today to join the thousands of students we've helped get into their top choice schools:

Trending Now

How to Get Into Harvard and the Ivy League

How to Get a Perfect 4.0 GPA

How to Write an Amazing College Essay

What Exactly Are Colleges Looking For?

ACT vs. SAT: Which Test Should You Take?

When should you take the SAT or ACT?

Get Your Free

PrepScholar

Find Your Target SAT Score

Free Complete Official SAT Practice Tests

How to Get a Perfect SAT Score, by an Expert Full Scorer

Score 800 on SAT Math

Score 800 on SAT Reading and Writing

How to Improve Your Low SAT Score

Score 600 on SAT Math

Score 600 on SAT Reading and Writing

Find Your Target ACT Score

Complete Official Free ACT Practice Tests

How to Get a Perfect ACT Score, by a 36 Full Scorer

Get a 36 on ACT English

Get a 36 on ACT Math

Get a 36 on ACT Reading

Get a 36 on ACT Science

How to Improve Your Low ACT Score

Get a 24 on ACT English

Get a 24 on ACT Math

Get a 24 on ACT Reading

Get a 24 on ACT Science

Stay Informed

Get the latest articles and test prep tips!

Follow us on Facebook (icon)

Samantha is a blog content writer for PrepScholar. Her goal is to help students adopt a less stressful view of standardized testing and other academic challenges through her articles. Samantha is also passionate about art and graduated with honors from Dartmouth College as a Studio Art major in 2014. In high school, she earned a 2400 on the SAT, 5's on all seven of her AP tests, and was named a National Merit Scholar.

Ask a Question Below

Have any questions about this article or other topics? Ask below and we'll reply!

Question Types on the AP World History: Modern Exam

April 9, 2024.

Question Types on the AP World History: Modern Exam

The AP World History: Modern exam is a 3-hour and 15-minute exam that tests your ability to think like a historian. It will ask you to analyze primary and secondary sources and identify patterns and connections that can support a historical interpretation. This AP exam will also test your knowledge of historical concepts covered in the AP World History: Modern course. Before taking the exam, review the types of questions you’ll encounter so you get comfortable with the format and know how to manage your time. Practice answering sample questions, like the ones below, and understand how the exam is graded, especially the free-response questions, to maximize your exam score. In this guide, we break down what you need to know about the four question types on the AP World History: Modern exam.

What are the Four Question Types on the AP World History: Modern Exam?

There are four types of questions on the AP World History: Modern exam, including multiple-choice questions, short-answer questions, and free-response (essay) questions. The free-response questions are composed of a document-based question (DBQ) and a long essay question (LEQ). In the first half of the exam, you will have 55 minutes to complete 55 multiple-choice questions. Immediately following, you’ll answer three short-answer questions in 40 minutes. The second half exam gives you 100 minutes to answer both the DBQ and LEQ. 

Multiple-Choice Questions on the AP World History: Modern Exam

The first part of the AP exam requires you to answer 55 multiple-choice questions in 55 minutes. This section of the exam is worth 40% of your total exam grade.

Each multiple-choice question on the AP World History: Modern exam includes four answer options; you will pick the one that BEST answers the question. These questions will be grouped in approximately fifteen to twenty clusters, typically of three or four questions. One point is awarded for each correct answer. Incorrect answers are not penalized, so answering all the multiple-choice questions is important even if you have to guess. 

Expert tip: Start by eliminating incorrect answers. Every distractor, or wrong answer, is supposed to sound somewhat plausible. Still, a quick but careful reading generally allows you to eliminate at least one wrong answer if not two. This is the first thing you should do. If you can quickly pick the correct answer from the two or three that remain, do so. If you can’t, flag the question and return to it during your second run through the exam.

[ LISTEN: Barron’s AP World History: Modern Podcast Episode 9: “Multiple-Choice Questions” on Apple and Spotify ]

Sample Multiple-Choice Question

The following is a sample multiple-choice question similar to one you might find on the AP World History: Modern exam. In this example, you must read the following excerpts and use them to answer the question below.

The story of my great-grandmother was typical of millions of Chinese women [before the 1911 revolution]. She came from a family of tanners. Because her family was not intellectual and did not hold any official post, and because she was a girl, she was not given a name. Being the second daughter, she was simply called “Number Two Girl.” [She never met her husband] before her wedding. In fact, falling in love was considered shameful, a family disgrace. Not because it was taboo, but because young people were not supposed to be exposed to situations where such a thing could happen, partly because it was immoral for them to meet, and partly because marriage was seen above all as a duty, an arrangement between two families. With luck, one could fall in love after getting married. 

from Jung Chang, Wild Swans: Three Daughters of China (1991)

It is a truth universally acknowledged, that a single man in possession of a good fortune must be in want of a wife. This truth is so well fixed in the minds of the surrounding families, that he is considered as the rightful property of some one or other of their daughters.

from Jane Austen, Pride and Prejudice (1813)

  • The observations made in the above quotations are best understood in the context of which of the following? a. The use of religious doctrine to regulate the role of women in society b. The impact of industrialization on family structures c. The oppression of women due to the rise of social Darwinist ideologies d. The shaping of marriage customs by prevailing social and economic norms

Check your answer.

Answer: (D) Contextualization and process of elimination help to answer this question. Neither quotation speaks directly to religion or industrialization, making A and B unlikely. While oppression is evident in the first quotation, it is not overtly so in the second, and has nothing to do with social Darwinism in any case, so C is incorrect. In both cases, but in different ways, the impact of social and economic factors on marriage is at stake, and those are what tie the questions together.

Short-Answer Questions on the AP World History: Modern Exam

After finishing the multiple-choice section, you’ll answer three short-answer questions. You will have 40 minutes to complete this section of the test, giving you roughly 13 minutes to answer each question. This section is worth 20% of your total exam grade. 

The short-answer questions will ask you to use content knowledge and various historical skills to provide written responses to three short-answer questions. You must complete the first and second questions; you will then have the choice to complete the third or fourth. 

  • Short-answer questions #1 and #2: These questions can cover any time period between 1200 and the present. The first question will require you to assess some sort of secondary source, and the second will test you on primary source material.
  • Short-answer questions #3 and #4: These questions will not provide any specific stimulus material. The third question will cover the period from 1200 to 1750, and the fourth will focus on the years between 1750 and the present. 

Each short-answer question will ask you to do three things, each of which will be worth one point. You are not required to develop a thesis for the short-answer questions. Your main strategy should be to complete all three questions within the set amount of time and to clearly indicate that, in each case, you have satisfactorily accomplished all three goals. Lengthy answers are unnecessary — one long paragraph, or perhaps two or three short paragraphs, should suffice.

[ LISTEN: Barron’s AP World History: Modern Podcast Episode 10: “Short-Answer Questions” on Apple and Spotify ]

Sample Short-Answer Question

Below is an example of a short-answer question similar to one you will encounter on the AP World History: Modern exam. Use the two passages below to answer all parts of the question that follows. 

It is nowadays common for Indian history textbooks to treat the various “empires” that successively occupied the stage of Indian history as so many successive repetitions with merely different names for offices and institutions that in substance remained the same: namely, the King, the Ministers, the Provinces, the Governors, and so on. But D.D. Kosambi, in his Introduction to the Study of Indian History, rightly observed that this repetitive succession cannot be assumed, and that each regime, when subjected to critical study, displays distinct elements. We know most, of course, about the Mughal Empire, which displays so many striking features. In its large extent and long duration, it had only one precedent, in the Mauryan Empire, some 1,900 years earlier. Some scholars regard it as the fulfillment of the political ambitions embodied in Indian polity for three millennia. And yet there is also a temptation to see in the Mughal Empire a primitive version of the modern state. Its existence belongs to a period when the dawn of modern technology had occurred in Europe, and some of the rays of that dawn had also fallen on Asia. Can it then be said that the foundations of the Mughal Empire lay in artillery, the most brilliant and dreadful representative of modern technology, as much as did those of the modern absolute monarchies of Europe?

M. Athar Ali, “Towards an Interpretation of the Mughal Empire,” 1978

The prevailing view of the Mughal Empire has been based on the mistaken assumption that this state was a kind of unfinished, unfocused prototype of the British Indian Empire of the late nineteenth and early twentieth centuries. A more fruitful approach is to treat the Mughal Empire as one example of the [older-fashioned] patrimonial-bureaucratic empire, featuring a depiction of the emperor as a divinely-aided patriarch, the household as the central element in government, members of the army as dependent on the emper – or, the administration as a loosely structured group of men controlled by the imperial household. It seems clear that to accept this interpretation of the empire is to accept the necessity of re-examining the entire structure of Mughal political activity.

Stephen P. Blake, “The Patrimonial-Bureaucratic Empire of the Mughals,” 1979

  • Using the excerpts above, answer (a), (b), and (c). a. Provide ONE piece of historical evidence (not specifically mentioned in the passage) that would support Ali’s interpretation of the Mughal Empire’s fundamental nature. b. Provide ONE piece of historical evidence (not specifically mentioned in the passage) that would support Blake’s interpretation of the Mughal Empire’s fundamental nature. c. Explain ONE way in which the views about Mughal governance expressed in the two passages led the authors to propose different interpretations of the empire’s fundamental nature.

Possible answers:

The scholarly debate at stake here is whether the Mughal Empire is best seen as the product of early political modernization—and a departure from earlier regimes ruling India—or as a government that followed older patterns of rulership. Ali promotes the first argument, while Blake, using the label “patrimonial” (in which the state is considered the personal property of a monarchical ruler), takes the second position. The easiest way to answer Part A is to mention the Mughal Empire’s success as a “gunpowder empire,” along with Ottoman Turkey, an adoption of modern technology that buttresses Ali’s thesis. One could also discuss the empire’s elaborate bureaucracy, much of which was in fact kept in place by the British as they extended their colonial reach over India. In favor of Blake’s argument in Part B, one could easily mention how religious policy—especially pertaining to Muslim rulership over India’s Hindu majority—varied widely based on the personal preferences of any given emperor, from Akbar’s remarkable tolerance to Aurangzeb’s extreme Islamic rigidity.

In explaining the differences between the two views, as Part C requires, it might be tempting to point to the two authors’ national differences, one being Indian, the other not. However, the debate does not seem to revolve around this question. Even though one might be able to say that patriotism inclines Ali to favor an argument depicting Mughal India as more modern than Blake appears to think, highlighting this sort of difference in this case runs the risk of stereotyping national viewpoints. (Still, be aware that national perspectives will sometimes affect debates of this type, especially when it comes to Western imperial treatment of other parts of the world, so in some instances it could be worth bringing up.) Perhaps the best way to answer this part of the question would be to point out that Ali seems mostly concerned with the Mughal Empire’s place as a particular stage in India’s long history, while Blake is mainly interested in the Mughal Empire as a political system typical of its era and in comparison with other regimes in Eurasia during a particular time.

Free-Response (Essay) Questions on the AP World History: Modern Exam

The free-response section of the AP World History: Modern exam lasts 100 minutes and consists of the document-based question (DBQ) and the long essay question (LEQ). The section begins with a 15-minute reading period, during which you are allowed to read both questions, examine the DBQ documents, and plan your responses (taking notes and making outlines). Alternatively, you can start writing immediately. Either way, you can write the essays in whichever order you wish, and you can use the time however you please. We suggest using the 15-minute reading period to read the documents and outline your answers. The rest of the time should be divided more or less evenly, with perhaps 45 minutes spent on the DBQ and 40 minutes on the LEQ.

A commonly followed guideline is to write the DBQ first. The documents will be fresh in your mind, and because the DBQ operates according to the most complicated rules, it will be good to have it out of the way. Just make sure to leave enough time for the LEQ! Practice writing essays in 40 minutes or less to improve your time management skills for this section.

The Document-Based Question on the AP World History: Modern Exam

Unlike the other essays, the DBQ on the AP World History: Modern exam asks you to perform well on two fronts. Not only does the essay itself have to be well written (complete with a good thesis), but you must demonstrate skillful handling of the seven documents. The DBQ is worth 25% of your total exam score. 

The DBQ will focus on some time period between 1450 and the present. When taken together, the documents address a particular theme or issue, typically with a fairly narrow focus when it comes to era, geography, and topic. For example, a DBQ might ask about industrialization in nineteenth-century Asia or European imperialism in a specific part of the world. Or it might ask about a noteworthy cultural trend, technological innovation, trade network, or sociological development. 

A DBQ will tie your use of the documents to one of several historical reasoning skills. You may be told to compare and contrast two things, to trace continuity and change over time, or to analyze causes and consequences. You will organize the documents into groups (typically three of them) and discuss the documents’ context as well as their creators’ point of view or purpose. To test your understanding of how documents can sometimes be of limited usefulness, the DBQ will ask you to identify additional evidence that, if provided, would shed further light on the question.

Expert Tip: Vague language equals a weak thesis! Be as specific as possible—but without getting swamped. Your thesis shouldn’t be too long or contain too much detail.

[ LISTEN : Barron’s AP World History: Modern Podcast Episode 11: “The Document-Based Question” on Apple and Spotify ]

Sample Document-Based Question

Use Documents 1–7 to answer the following sample document-based question. 

  • Evaluate the extent to which Pan-Arabism and Pan-Africanism have differed in their nature and political effectiveness in the modern era.

long essay question ap world example

During the global wave of decolonization that followed the end of World War II, two political ideologies seemed poised to gain permanent prominence: Pan-Africanism and Pan-Arabism. Both movements followed the same logic, attempting to empower previously colonized peoples by encouraging a sense of unity based on broad cultural and ethnic identities, rather than on narrow national ones. A key difference, however, is that Pan-Africanism defined itself mainly in terms of opposition to racial and colonial oppression, whereas Pan-Arabism was in a better position to appeal to a common linguistic and religious heritage. In the end, while both achieved certain successes, both proved weaker than the lure of traditional nationalism and fell far short of their original aspirations.

Pan-Africanism had deeper roots than Pan-Arabism, and much of it sprang from the discontent felt by African-descended populations who lived among abusive white majorities or under white colonial rule. These experiences are spoken of by the 1920 “Declaration of the Rights of the Negro Peoples of the World” (document 4) and the 1945 poem “In Memoriam,” by Leopold Senghor (document 7). Senghor, from the French colony of Senegal, describes the alienation he feels living in Paris, surrounded by French whites—supposedly his “brothers,” but completely different, and to be feared, with their “faces of stone,” “blue eyes,” and “hard hands.” The contrast between his native landscape and holidays with the streets of Paris and the Catholic Day of All Saints reminds him that he does not belong. As a central figure in the negritude movement, Senghor stressed African identity as a source of strength for all those of African descent living under white rule, no matter where or under which colonial authority. The same viewpoint was expressed even more forcefully by the delegates to the 1920 Convention of the Universal Negro Improvement Association in New York City. The idea here was that all those of African descent shared a common problem— white oppression, whether in the United States, the Caribbean, Europe, or Africa itself under colonial rule—and a common heritage, and should therefore band together, regardless of language or specific tribal origin. The Convention’s “Declaration” went so far as to nominate an actual president of Africa, and to designate a song about Ethiopia as its anthem.

Document 2, an essay from the 1960s by Tanzanian president Julius Nyerere, carries the Pan-African vision into the post-World War II era, when many African nations overthrew colonial regimes or were in the process of doing so. Along with Kwame Nkrumah, who won freedom for Ghana from British rule, Nyerere was one of this time period’s most outspoken supporters of Pan-Africanism. (Also like Nkrumah, Nyerere can be viewed in the larger context of “Third World” political leaders such as Sukarno of Indonesia, Nehru of India, and Nasser of Egypt, all of whom sought ways to strengthen their newly liberated countries and keep them free of Western political interference and economic domination.) Nyerere identifies a key “dilemma” of Pan-Africanism: the fact that African independence has to be achieved country by country, after which it is tempting for each country to focus narrowly on its own interests. He argues vehemently that Africa can prosper only if such temptations are overcome in favor of a larger, more cooperative understanding of what it means to be African. The mural pictured in document 5 is a visual representation of this utopian ideal, which seeks to unite a billion people under one inclusive label, bridging all other ethnic or linguistic differences. Neither document, however, sufficiently addresses the huge difficulties involved with uniting peoples as diverse as those in Africa actually are. While politicians like Nyerere and Nkrumah managed to establish transnational organizations like the Organization of African Unity (now the African Union), conflict and disunity have tended to greatly outweigh Pan-African unity in the decades since World War II. Differences in language, ethnic and tribal identity, and local culture and religion seem to have proven stronger than abstract ideology.

Documents 1, 6, and 3 all have to do with Pan-Arabism. Gamal Nasser, who had just risen to power in Egypt as a nationalist leader, and would outrage much of the West by establishing Egyptian control over the Suez Canal, ranks with Nyerere as a major political figure in the “Third World” during the 1950s and 1960s. Like the Pan-Africanists, Nasser saw Pan-Arabism as a means to build strength and defy the militarily stronger and more economically prosperous nations of the West. To that end, he even managed to join Egypt to Syria and Iraq in a short-lived United Arab Republic. Much more so than Pan-Africanism, Pan-Arabism had the potential to foster transnational unity, because most people in the so-called Arab world shared a common language (Arabic), a common history and cultural heritage, and, in most cases, a common religion (Islam). Document 6, a recent newspaper editorial, makes this exact point in calling for a revival of the Pan-Arab ideal in order to preserve Arab identity in the face of westernization. However, the fact that document 6 has to call for Pan-Arabism in 2012 clearly indicates that, even with the advantage of a common tongue and heritage, Pan-Arabism—like Pan-Africanism—yielded few concrete political results. Some of the region’s smallest nations may have fused together as the United Arab Emirates, but this is hardly a political powerhouse. Document 3, a graphic showing the “Arab world,” with each nation linked to its individual flag, does not say anything overtly—but by its very nature speaks to the failure of Pan-Arabism to overcome national differences in favor of a larger regional identity. Like Nkrumah and Nyerere, Nasser failed to cement in place a larger dream of union.

As rich as the current selection of documents is, you should include additional perspectives to shed even more light on this comparison between Pan-Africanism and Pan-Arabism. While political elites are well represented, the point of view of nonstate actors is not, and there is much historical evidence to show that the concerns of most ordinary people tend to be local or national, and rarely transnational. Even political leaders who theoretically supported Pan-Arabism, like Gaddafi in Libya and Nasser himself, cared more for their own nations’ interest in the end. With respect to Africa, numerous episodes—such as the Biafra War (in which the Igbo minority tried to secede from Nigeria in the late 1960s) and the 1994 genocide committed in Rwanda by the Hutu against their Tutsi neighbors—provide evidence of the real-life difficulties involved with realizing the dream of Pan-African unity. Finally, to contextualize this topic even further, it would be useful to compare it with trends in Asia during this period. There, too, especially in South and Southeast Asia, many nations decolonized, and a number of them tried to form transnational coalitions or alliances. While Pan-Asianism was not attempted to the same degree that Pan-Arabism and Pan-Africanism were, organizations such as the Southeast Asian Treaty Organization (SEATO) and the Association of Southeast Asian Nations (ASEAN) emerged to accomplish at least some of the same things that Pan-Africanists and Pan-Arabists hoped to bring about in their own regions.

How is the Document-Based Question Scored?

The table below shows the official AP scoring guide for the DBQ. The maximum score you can receive is a “7.”

Scoring table for the document-based question (DBQ) on the AP World History: Modern exam.

The Long Essay Question on the AP World History: Modern Exam

The AP World History: Modern exam’s long essay question (LEQ) asks you to develop and support an argument based on evidence. Although you can write this essay before the DBQ if you wish, we recommend you save it for last because of the DBQ’s complexity. Aim to spend about 40 minutes on the LEQ.

You’ll choose between three LEQs. Be sure to select the one you feel the most confident in answering. All three options will test the same historical reasoning skill and focus on the same course theme but cover a different time period: the first will cover 1200–1750, the second will focus on 1450–1800, and the third will do the same for 1750 to the present. The reasoning skill will vary from year to year, rotating through causation, comparison, and continuity/change over time.

[ LISTEN: Barron’s AP World History: Modern Podcast Episode 12: “The Long Essay” on Apple and Spotify ]

Sample Long Essay Prompt

Below are three sets of questions, each reflecting how the AP exam, in any given year, might target a particular historical reasoning skill and course theme. One sample answer—responding to one of the comparison questions—is provided.

Directions: Answer Question 1 or Question 2 or Question 3.

Possible Comparison Questions (Course Theme = State Building)

  • In the period 1200 to 1500, states in Europe and in India used various techniques of conquest and rulership to consolidate and centralize their authority. Develop an argument that evaluates the extent to which states in each region succeeded in their goals of political consolidation and centralization.
  • In the period 1450 to 1800, the Ottoman Empire and China employed various strategies to legitimate their political authority. Develop an argument that evaluates the extent to which each state succeeded in this goal. 
  • In the period after 1900, both Latin America and the Middle East significantly modernized their political systems. Develop an argument that evaluates the extent to which this process was accomplished in each region.

Check your answer to Question #3.

The first half of the twentieth century brought immense changes to many parts of the globe. Among these was increased modernization in a number of non-Western regions, including Latin America and the Middle East. Both places faced similar obstacles, such as relative socioeconomic backwardness, heavy influence from outside powers, and limited success in the past with representative government. There were, however, key differences as well. Latin American states, with their generally longer history of independence, tended to be more modern already. Traditional religion was less of a barrier to change in Latin America than in the Middle East, and events like the world wars and the Great Depression affected both areas differently. On the whole, these differences outweighed the similarities, causing Latin America to make more progress toward modernization than the Middle East.

At the beginning of this period, both Latin America and the Middle East suffered from social and economic underdevelopment. Wealthy elites, whether colonial, corporate, or royal and aristocratic, benefited from the unbalanced exploitation of a handful of commodities. In Latin America, these included foodstuffs (beef, coffee, bananas, and other fruit) and natural resources such as copper, steel, fertilizer, and, in some countries, oil. Oil was even more central to the economies of the Middle East, just as it is today. This “banana republic” overexploitation of resources discouraged the healthy diversification of economies and kept societies rigidly stratified, with small upper classes dominating large, impoverished majorities. Although some of this changed between 1900 and 1945, it limited modernization throughout the period.

Another similarity is that Latin American and Middle Eastern states tended to be heavily influenced by outside powers, both before and during this half century. Prior to World War I, much of the Middle East was ruled by the Ottoman Empire or fell into European spheres of influence—and even though the Ottomans fell after WWI, European spheres of influence grew even larger when the post-WWI mandate system placed much of the Middle East, especially the Ottomans’ former Arab possessions, under French and British custody. Most Latin American states had been free since the wars of independence of the early 1800s, but foreign investors (like America’s United Fruit Company) wielded much power over Latin American governments, and the U.S. government regarded the region as part of its political sphere of influence. The Pan- American Union and even Franklin Roosevelt’s Good Neighbor Policy were instruments of U.S. diplomatic interests throughout Latin America. Even when these outside interests did not deliberately oppose modernization (and they often did), it was rarely in their interest to actively support economic diversification, and because dealing with cooperative elite classes was easier than negotiating with elected governments representing a range of popular interests, they did not necessarily support democracy either.

On the other hand, important differences moved Latin America further down the path toward modernization. To begin with, Latin American states, while nonindustrialized by the standards of Western Europe and North America, had undergone more industrialization than most parts of the Middle East. With this kind of foundation to build on, countries like Mexico and Argentina, for example, found it easier to create sizable industrial sectors after WWI. Latin American states also had a tradition of constitutional rule stretching back to the era of Simo'n Bol var in the early 1800s, and while those constitutions were not always perfectly followed, they created a more favorable climate for progress when it came to gender equity, enlarging the middle classes, and reforming electoral systems. Less of this was possible in the Middle East.

Another crucial difference involves the role of religion. Although Catholicism was overwhelmingly central to Latin American culture, and although it tended to exert a conservative influence over public life there, by this point in history it was not nearly as much a barrier to social and economic progress as traditional Islam still was in the Middle East. It is no coincidence that the Middle Eastern states that modernized most were the small handful where energetic westernizing autocrats—most famously Mustafa Kemal Ataturk of Turkey and Reza Khan Pahlavi of Persia (Iran)—defied the will of Muslim clerics, secularized their states, industrialized their economies and educational systems and, in Ataturk’s case, gave women the vote. By the early twentieth century, such fierce conflict with institutional religion was far less necessary for Latin America to modernize. In the Middle East, by contrast, it remains a struggle even today to balance modernization with respect for Islamic tradition.

Another comparison to consider is the prevalence of dictatorship in both regions during these years. Whether they were monarchs or autocratic strongmen, authoritarian leaders in both regions were often the agents of modernizing change. In the Middle East, the primary impulse for economic and social modernization was typically the will of determined authoritarians, such as the above-mentioned Ataturk and Pahlavi. Among the Latin American dictators who promoted industrialization and other modernizing changes were the Perons in Argentina and the Vargas government in Brazil. One last point that calls out for attention is the pronounced trend toward authoritarian rule worldwide during this period. A truly contextual view of this topic must take into account the rise of dictatorial or nondemocratic regimes in places like Asia (Japanese militarism during the 1930s, warlords and Chiang Kai-shek in China after the collapse of Sun Yat-sen’s democratic revolution) and Europe (most notably Lenin and Stalin in Russia, Mussolini in Italy, and Hitler in Germany). As with the regions discussed more directly by this question, dictatorship in these other places arose in some cases due to political and economic crises like the Depression, and in other cases due to pressures brought about by rapid modernization.

How is the Long Essay Question Scored?

The LEQ is less complicated than the DBQ, but certain tasks need to be accomplished the right way to earn a maximum score of six points. Below, we break down the scoring guide for the AP World History long essay question.

Scoring table for the long essay question (LEQ) on the AP World History: Modern exam.

AP Biology Resources

  • About the AP Biology Exam
  • Top AP Biology Exam Strategies
  • Top 5 Study Topics and Tips for the AP Biology Exam
  • AP Biology Short Free-Response Questions
  • AP Biology Long Free-Response Questions

AP Psychology Resources

  • What’s Tested on the AP Psychology Exam?
  • Top 5 Study Tips for the AP Psychology Exam
  • AP Psychology Key Terms
  • Top AP Psychology Exam Multiple-Choice Question Tips
  • Top AP Psychology Exam Free Response Questions Tips
  • AP Psychology Sample Free Response Question

AP English Language and Composition Resources

  • What’s Tested on the AP English Language and Composition Exam?
  • Top 5 Tips for the AP English Language and Composition Exam
  • Top Reading Techniques for the AP English Language and Composition Exam
  • How to Answer the AP English Language and Composition Essay Questions 
  • AP English Language and Composition Exam Sample Essay Questions
  • AP English Language and Composition Exam Multiple-Choice Questions

AP Human Geography Resources

  • What’s Tested On the AP Human Geography Exam?
  • AP Human Geography FAQs
  • AP Human Geography Question Types and Strategies
  • Top 5 Study Tips for the AP Human Geography Exam

FOLLOW ALONG ON SOCIAL

AP World History: Modern

Learn all about the course and exam. Already enrolled? Join your class in My AP.

Not a Student?

Go to AP Central for resources for teachers, administrators, and coordinators.

About the Exam

The AP World History: Modern Exam will test your understanding of the historical concepts covered in the course units, as well as your ability to analyze primary and secondary sources and identify patterns and connections that can support a historical interpretation.

Thu, May 8, 2025

12 PM Local

AP World History: Modern Exam

This is the regularly scheduled date for the AP World History: Modern Exam.

Exam Components

Section ia: multiple choice.

55 questions 55mins 40% of Score

The questions in the multiple-choice section come in sets of usually 3–4 questions based on the same stimulus. The questions will include one or more sources to respond to such as primary and secondary texts, images (for example, artwork, photos, posters, cartoons), charts, and maps.

You'll be asked to:

  • Analyze the provided sources
  • Analyze the historical developments and processes described in the sources

Section IB: Short Answer

3 questions 40mins 20% of Score

In the short-answer section, you’ll write answers to questions in your test booklet. Some questions include texts, images, graphs, or maps.

  • Question 1  is required, includes 1 or 2 secondary sources, and focuses on historical developments or processes between the years 1200 and 2001.
  • Question 2   is required, includes 1 primary source, and focuses on historical developments or processes between the years 1200 and 2001.
  • You can choose between  Question 3   (which focuses on the period from 1200 to 1750) and  Question 4   (which focuses on the period from 1750 to 2001) for the last question. No sources are included for either  Question 3  or  Question 4 .
  • Analyze historical developments and processes described in the sources
  • Put those historical developments and processes in context
  • Make connections between those historical developments and processes

Section II: Free Response

2 questions 1hr 40mins 40% of Score

In the free-response section, you'll write answers to questions in your test booklet. There are two questions: one document-based question and one long essay.

Document-Based Question Recommended Time: 60 Minutes (includes 15-minute reading period) | 25% of Score The 60-minute recommended time for this section includes a 15-minute reading period.

  • You'll be presented with seven documents that give various perspectives on a historical development or process.
  • You’ll be asked to develop and support an argument based on these documents and other evidence from your own knowledge.
  • The topic of the document-based question will include historical developments or processes between the years 1450 and 2001.

Long Essay Question 40 Minutes | 15% of Score

  • You'll have a choice of three questions; you’ll pick one to answer.
  • Each tests the same skills but the questions focus on different historical time periods (either the period from c. 1200–1750, from c. 1450–1900, or from c. 1750–2001).
  • You'll be asked to develop and support an argument based on evidence.

Exam Essentials

Exam preparation, ap classroom resources.

Once you join your AP class section online, you’ll be able to access AP Daily videos, any assignments from your teacher, and your assignment results in AP Classroom. Sign in to access them.

  • Go to AP Classroom

Free-Response Questions and Scoring Information

Go to the Exam Questions and Scoring Information section on the AP World History: Modern Exam page at AP Central to review the latest released free-response questions and scoring information.

Past Exam Free-Response Questions and Scoring Information

Go to AP Central to review free-response questions and scoring information from past AP World History: Modern exams. 

AP World History: Modern Course and Exam Description

This is the core document for the course. It clearly lays out the course content and describes the exam and the AP Program in general.

Services for Students with Disabilities

Students with documented disabilities may be eligible for accommodations for the through-course assessment and the end-of-course exam. If you’re using assistive technology and need help accessing the PDFs in this section in another format, contact Services for Students with Disabilities at 212-713-8333 or by email at [email protected] . For information about taking AP Exams, or other College Board assessments, with accommodations, visit the Services for Students with Disabilities website.

Credit and Placement

Search AP Credit Policies

Find colleges that grant credit and/or placement for AP Exam scores in this and other AP courses.

Additional Information

All Subjects

Comparison in the AP Histories

13 min read • june 18, 2024

Evan Liddle

Evan Liddle

📘College Board Description

In short a student will need to be able to do the following: 

  • Describe similarities and/or differences between different historical developments or processes.
  • Explain relevant similarities and/or differences between specific historical developments and processes.
  • This one basically means, you as a student will need to make a judgment about the importance of certain developments or processes.  Compare the importance of events.

🔎Organizing Question

How are places, events, and developments both similar and different? And why are these both similar and different?

Think about an apple and an orange. You can probably think of many  differences . Color, taste, size, shape and smell. Both are round, but people generally eat the skin of apples whereas most people do not eat the skin of oranges. 

Comparison enables one to explore the context of various parts of the world as it was and is. This contextualization is an important part of comparing or else looking at two parts of the world would be a fruitless exercise as we would have no idea why these things are different. From politics and culture to economics and technology, the world continues to be a very diverse place, and being able to effectively contextualize and compare parts of it is critical.  

Back to the fruit, however, what about the  similarities ? Initially Apples and Oranges don’t seem so similar, however look close. They are both fruit, and they are both roughly circular, they are both considered healthy to eat. And this is perhaps the tougher part of comparison, finding the underlying similarities between two places that seem obviously different.   

The following are brief summaries of comparison per AP World History’s time periods. As you read, consider the context of each situation and practice coming up with additional ways to compare these topics.  

🎥Watch: WHAP -   Introduction to Historical Reasoning

Unit 1 Comparisons

Short but sweet: Unit 1 focuses on the variety of political states so the college board cites political comparison among the types of states seen during this time.

Below is a breakdown of the major government types witnessed in Unit 1. 

The original Islamic form of government, a combination of classic Imperial Rule and Islamic principles. The Caliph was the leader of the whole Muslim community but also the ruler of a large empireSee the Abbasid Caliphate  Different from a Caliphate in that while the leader was Muslim, they made no claim to religious leadership, many Sultans paid lip service to the existing Caliph though they did not submit political power.See the Delhi Sultanate or the Mamluk Sultanate    While these states nominally had an all powerful king, for the most part local lords ran the show and used peasant labor to pay their knights and exercise control. Often, these rulers had more influence over the population than a king or queenSee both Medieval Europe and Medieval Japan
This applied only to China. Specifically the Song Dynasty and later Yuan Dynasty. These dynasties relied on the connection (Song) or appearance of connection (Yuan) to Confucian Values, the Imperial Bureaucracy, and the concept of the Mandate of Heaven.  See the Song Dynasty or Yuan Dynasty These were the slew of new states in South and Southeast Asia between 1000-1400. These states relied on connections or claims to incarnation with Hindu Gods or Buddhist rituals, sometimes both, for authority with the people. See the Srivijaya Empire, or Khmer Empire   These are cities and some land around them which sit around major trade routes, giving them a power to shape the course of trade. They often were the site of major cultural interactions, and almost all major religious and cultural groups could be found here.See Melaka, or the Swahili City States

When comparing these types of governments, ask yourself about why these types of governments exist? What role does  Geography play? What role does  Culture play? What role does  Trade or  Economics play?

For example, the Chinese Imperial State is similar to a Caliphate in that the rulers of both have absolute authority over their people. However, their justification for ruling is different. The Caliphate relied on the authority of Islam, being an Islamic State, whereas China relied on a Confucian Hierarchy and the Mandate of Heaven. 

Unit 2 Comparisons

Short but sweet: Unit 2 focuses on trade networks and networks of exchange so the comparisons focus on the new v. old networks, and within existing trade networks.

| | Silk Road Trade Network (Old) | Indian Ocean Trade Network (old) | Trans-Saharan Trade Route (New) | | --- | --- | --- | --- | | Commercial Practices   | Bills of Credit (China) Banking (Islamic World). | Hindu Temples as banking institutions. | State support of Merchants.  | | Role of Technology   | Paper Money and Caravansaries  | Dhow ships and knowledge of Monsoon Winds | Use of Camels and Camel saddles | | Cultures spread | Islam to Asia, Number System to Europe  | Hinduism and Buddhism to Southeast Asia | Islam to West Africa | | Goods traded | Porcelain, Precious Stones, Silk | Spices, Gold, Ivory, Textiles, Sugar, Silk, Porcelain  | Salt, Slaves, Gold, Iron Products | | Impact on the Environment | Spread of Citrus Fruit and Sugar in the Islamic World | Spread of Rice Varieties in East and Southeast Asia  | Introduction of Camels to West Africa |

Which trade routes expanded during this time? Well the Silk Road did not so much expand, but thanks to the Mongols it was used more, meaning the use of it  intensified . The Trans-Saharan Route was new so that was a trade route  expansion .

Notice: all the trade routes trade predominantly  Luxury Goods .💎 This is a feature of all trade routes until modern times. The risk of trade meant merchants needed high value products to make the journey worth it. 

Unit 3/4 Comparisons

Short but sweet: Unit 3 and 4 focus on the growth of empires both across the Atlantic in the Americas and in Afro-Eurasia, so the major comparison would be in the type or style of government. 

Mughal EmpireOttoman EmpireSafavid EmpireSonghai EmpireRussian Empire Qing Empire (China)Ming Empire (China)Incan EmpireMexica (Aztec Empire)Tokugawa JapanBritish EmpireSpanish EmpirePortuguese EmpireFrench Empire Dutch Empire

A major difference between these two types of empires is that the empires on the left were predominantly land based, meaning most of their power and wealth came from the land, rich agriculture , and  profitable trade routes . Those on the right are predominantly maritime based empires, meaning that their power and wealth come from  trade overseas,  colonies overseas, or controlling  overseas trade routes .   

Administrative Systems

Divine Right of Kings (French Empire)Mandate of Heaven (China)Songhai Islam (Songhai)Devshirme System (Ottoman Empire)Samurai Warriors (Japan)

How governments maintain legitimacy varies across empires varies, but the chart above illustrates two general ways that empires claimed the right to rule.  Military elites refer to a system of warriors who are loyal to the ruler or the state which helps them maintain power. On the other hand some states relied more on  religious justification for their power, claiming direct connection to the divine as the right to rule. It should be understood, these two often mixed.  

Social Hierarchies

Qing Dynasty (Restrictive Policy on Han Chinese)Spanish Empire (The Casta System)The Ottoman Empire (The Millet System)The Mughal Empire (Zamindar/Rajput System)

Organizing a society is essential to maintaining stability within an Empire. A hierarchy determined one's  job , what  legal rights they had and  proximity to power. These hierarchies can be based on physical characteristics, probably the most famous being the  Casta System , which was based on perceptions of blood purity. Or they can be based on religion as in many of the Islamic empires; the  Millet System  gave each religion its own political zone. Although the Islamic zones were above the others, this did provide some stability as each zone was free to practice their own religion.   

Empire Type (Maritime Empires)

The Portugese in Africa/IndiaThe Dutch in Southeast AsiaThe French in North AmericaThe British in North AmericaThe Spanish in North and South America

For those empires that expanded overseas, there were two major types that corresponded more to the situation of the colonized regions. During this time, Europeans stuck to themselves in trading posts and did not assert authority over land, but did overseas known as  Trading Post Empires .  Colonial Empires refers to places where Europeans conquered land and sometimes settled their own populations there. Trading Post Empires often confronted established and long standing empires or populations they could not eliminate or geography that was unfavorable to conquest. The opposite was true for Colonial Empires.

Unit 5 Comparisons

Short but sweet: Unit 5 focuses on Industrialization. The major comparisons are on nations with industrialization and those nations which de-industrialized in response to expanding European economic power.  

Industrialization Outside of Europe

   
The Abolition of Serfdom (1861) created an Industrial Labor Force, but did not abolish the landowning classImporting technical expertise from Europe created first factories Initial industry focuses on heavy industry specifically railroads and steel production Intensified control over Siberia as a market and location of resourcesHeavy government involvementBy 1900 Russia was semi-IndustrializedThe Meiji Restoration (1868) reset the political structure of Japan. The feudal landowning class was abolished  Importing legal and technical expertise created their first factories Initial industry focuses on Japan’s silk textile industry and later on producing weaponsImperial expansion in Korea and Taiwan to support economyHeavy government involvementBy 1900, Japan was heavily industrializedSome Industry before Civil War, but victory of the North over slavery (1865) meant Industry would be dominant politically over farming Long history of inventions and developments means no need to import expertiseInitial industry focuses on cotton textiles, later spreading to many other industriesConquest of western North America to create food supply and ideological belief (Manifest Destiny)Some government involvementBy 1900 the United States was heavily industrialized

Industrial Decline in Response to European Industrialization

 
British Textiles drowned the traditional market for Egyptian fabrics, putting many producers out of business by 1900.In response, Egypt became a source of Cotton for the British Textile Industry.British Textiles drowned the traditional market for Indian fabrics, putting many producers out of business by 1900.In response, Egypt became a source of Cotton for the British Textile Industry.The rise of steel ships eliminated the shipbuilding yards in South Asia that had served the East India Company.

Unit 6 Comparisons

Short but sweet: Unit 6 is about the expansion of the European and American Empires and a comparison in this unit could focus on the types of empires created during this time, much like Unit 3/4.  

State Expansion in the Age of Empire

 
New Zealand White Highlands in KenyaWestern United StatesFrench Expansion in  North Africa British Expansion in West AfricaCongo Free State to Belgian GovernmentDutch East India Company to Dutch Government

The Age of Imperialism, which is different from the Age of Colonialism (Unit 3/4) in the expansion of European States, involved European States asserting both political and economic control over the planet.

State Expansion varied by region. In those places where the geography and demographics were favorable  Settler Colonies were either established directly by governments or by waves of immigration. These states eliminated or displaced the location populations and replaced them with the state’s own population. However, in cases where the populations were too large or state control was not as strong, direct state expansion simply incorporated a conquered region into the state itself, the traditional definition of Imperialism. During Unit 3/4 several major private companies built large colonial empires. However, by Unit 6 these companies were bankrupt or their states had grown strong enough to assume control, or in the case of the Congo, a major scandal led to the incorporation of these territories into their respective empires.      

Unit 7 Comparisons

Short but sweet: Unit 7 Focuses on global warfare, specifically World War I and World War II, so any comparison will focus on global war or the time in between the conflicts 

Government Responses to the Great Depression

   
German government makes deals with Industrial Companies to develop certain products and keep prices low.The  and the  as products that were designed to be affordable for citizens. Having these items would improve Government legitimacy in the eyes of the German People.The Mexican and Brazilian Governments took the lead in developing specific industries to  make their nations economically stronger.  creating PEMEX, Mexican Petroleum to raise money for development to fund the construction of steel and autowork factories.The United States adopted Social Democratic measures to counteract the Great Depression.U.S. President Franklin Delano Roosevelt’s government created  to alleviate suffering from the Depression and attempt to kickstart the economy. 

### Five Year Plans in the Soviet Union

Though cut off from most international trade, the Soviet Union undertook a project of  rapid internal industrialization that was controlled and directed by the state to build up arms and manufacturing industries🏭.  The First Five Year Plan (1928-1932,  I know, not five years ) resulted in the almost full industrialization of the USSR faster than any nation had industrialized to this time in history. 

Government Conduct of World War II

Governments took control of arms production to ensure supplies for the military Governments used Ideology and Propaganda to gain support for the WarThe need for the Soviet Union to defeat Fascism and spread the ***Worldwide Communist Revolution.***The need for Nazi Germany to defeat the Soviet Union for  Governments sold war bonds and made contracts with private companies to ensure supplies for the military France, Britain, and the United States utilized their colonial holdings to help fight the Axis Powers.The Democratic nations made claims aspiring to self determination and democracy for all. 

Unit 8 Comparisons

Short but sweet: Unit 8 Focuses on the World from 1945-1980s, involving mainly the Cold War and the major impacts of decolonization around the World. Comparisons in this unit officially focus on decolonization methods and military action in the Cold War.

Gaining Allies during the Cold War

 
Military Alliances  Aid Support for newly Independent Nations      Military Alliances Support for Anti-Communist GovernmentsS. Vietnam 

The Cold War is defined as a major political rivalry between the United States and the Soviet Union that lasted from the End of WWII to 1989 (1945-1989). During this time both sides attempted to gain allies and influence nations to join their respective ideologies. Military alliances and military aid were a major factor. Though the major military alliances (Warsaw Pact v. NATO) never went to war, many small conflicts were fought between and within nations, proxy wars where each side was backed by either the U.S. or the U.S.S.R.. Though it should be noted, not all of these conflicts were specifically fought by communists v. anti-communists. For example, the Arab-Israeli Wars were not about communism, but each side was largely backed by one super power.

Methods of obtaining independence.

India (From Great Britain)The Gold Coast (From Great Britain)The Philippines (From the United States)Vietnam (From France & the U.S.)Algeria (From France)Angola (From Portugal)

The Cold War was also the Age of Decolonization, the dismantling of the major colonial empires from Unit 3, 4, and 6. Depending on the situation, this may have been violent or peaceful. If the colonial power in question had a large military, such as France or a strong ideological attachment to their colonies, such as Portugal, then the conflict was more likely to involve an armed struggle💪.

This is another example of the proxy conflicts mentioned earlier, the Soviet Union largely encouraged decolonization whereas the U.S. wanted to keep their allies powerful, which meant quietly downplaying decolonization in the 1950s-1970s.  Angola is an excellent example, where several groups (backed by either the U.S. or Soviet Union) fought Portugal, and each other, until a revolution in Portugal ended their empire.

However, in other cases, decolonization could be (relatively) peaceful. If the colonial power was too exhausted by World War II or if the colony had strong pre-existing independence movements.  India is an excellent example. India had a long standing politically organized independence movement. The Indian  National Congress and  Muslim League  and Great Britain was largely exhausted from WWII, and so India (and Pakistan) achieved independence without armed struggle against the colonizer.      

Unit 9 Comparisons

Short but Sweet: Officially there is no required example of comparison for Unit 9 as it focuses on the changes and continuity of the 20th century. However, here is one example of topics that can be compared. 

Diseases and Illnesses since 1900. Diseases and Illnesses  since 1900. 
    Rubella    **Diabetes Animal Flus (Bird Flu, Swine Flu et all)**

Facts to note about this is that the eliminated diseases often were eradicated or reduced because of  vaccines or increased access to medical care or  better living conditions in general such as clean water or proper sanitation. 

Facts to note about the diseases on the rise are that these are connected to an increasingly  interconnected world . 

For example, Ebola and various Animal Flus were previously confined to their origin points, and the other diseases such as Alzheimers are the product of humans living longer than before, which leads to problems with the function of organs. 

On a more contemporary note, the worldwide spread of  Covid-19 in the year 2020, represents both of these trends perfectly. Comparing the nature of diseases in the 21th century would be one way to practice comparison in Unit 9.

© 2024 Fiveable Inc. All rights reserved.

Ap® and sat® are trademarks registered by the college board, which is not affiliated with, and does not endorse this website..

long essay question ap world example

  • AP Calculus
  • AP Chemistry
  • AP U.S. History
  • AP World History
  • Free AP Practice Questions
  • AP Exam Prep

AP US History Long Essay Example

AP U.S. History Long Essay Example

The second part of Section II of the AP exam contains three long essay questions—you must respond to one. The AP U.S. History long essay question assesses your ability to apply knowledge of history in a complex, analytical manner. In other words, you are expected to treat history and historical questions as a historian would.

This process is called historiography—the skills and strategies historians use to analyze and interpret historical evidence to reach a conclusion. Thus, when writing an effective essay, you must be able to write a strong, clearly developed thesis and supply a substantial amount of relevant evidence to support your thesis and develop a complex argument.

The College Board’s characteristics of a high-scoring long essay question response are listed below. Note that the requirements are very similar to those of the DBQ; the primary difference is that any requirements related to use of the documents are removed from the scoring requirements for the long essay question.

[ RELATED: Reading Strategies for AP US History Exam ]

Long Answer Sample Question

Evaluate the extent to which the migration of European colonists and the resulting encounters with American Indians affected social patterns in the period from 1495 to 1650.

Step 1: Analyze the Prompt

As you choose which question you will answer, begin thinking about what your thesis will entail and how your essay will demonstrate a complex understanding. The notes of a sample high-scoring writer are below.

Thesis (with complex understanding): Spanish, French, and British each used territory differently; result: distinct social patterns

This writer claims that three different countries’ approaches to settling territories resulted in different types of social development, but other types of thesis claims are possible. For instance, the thesis could make a single claim about overall social patterns (such as “Europeans’ economic goals in the New World and their attitudes of superiority over American Indians resulted in exploitative and hierarchical social structures”) or focus on only one or two European nations.

Step 2: Plan Your Response

  • Context : motives for European exploration: new technology, navigation techniques, and trade routes
  • Thesis   (with complex understanding ):  Spanish, French, and British each used territory differently; result: distinct social patterns
  • goals: wealth and spread Catholicism
  • methods: mining, large-scale agriculture, encomienda, disease/weapons, missions
  • results: forced assimilation, social structure
  • goal: fur trade
  • method: mutually profitable trade relationships
  • result: alliances
  • goals: permanent settlements, Jamestown, religious freedom (New England)
  • methods: occupying more land for farming, smallpox, Metacom’s War
  • results: deaths of indigenous populations
  • ¶ conclusion: where Europeans sought permanent settlements or forced labor, resulted in American Indian population decline, upheaval, and threats to tradition

Step 3: Action! Write Your Response & Step 4: Proofread

See the following high-scoring response, and be sure to read the rubric to help you identify what makes this response effective. Think about what features you can incorporate into your own free- response answers.

Sample High-Scoring Response

The Spanish had two major goals: to gain wealth and to spread Catholicism to the native populations. Realizing the potential to mine precious metals and profit from large-scale agriculture, the Spanish forced American Indians into labor, such as through the encomienda system. Violence and deception were often used to subdue the indigenous populations, aided by the technological superiority of European weapons and the spread of devastating diseases. Although some Spanish came as missionaries with the goal of converting American Indians to Christianity and often protested the abusive treatment of the American Indians, even missions sometimes essentially forced labor and coerced assimilation to Spanish culture. In the long term, a hierarchical social structure developed in the Spanish colonies in which the Spanish-born and their descendants (peninsulares and creoles) dominated those of mixed background (mestizos and mulattos) and especially those of pure African or American Indian heritage. Overall, millions perished between disease and mistreatment, devastatingly weakening traditional cultures but enriching the Spanish.

The French differed from the Spanish in their relationship with the indigenous populations. Using the St. Lawrence River for transportation and trade, the French profited from trading fur pelts, particularly beaver, with the American Indians, and then sending the pelts to Europe. These traders profited from the knowledge and goods of the American Indian populations who lived there, and certainly desired to develop mutually profitable relationships with them. Overall, this more cooperative relationship helped preserve American Indian cultures and led to alliances between the French and different American Indian nations. These alliances benefited the French in later wars with the British.

You might also like

APUSH Period 8 Study Notes

Call 1-800-KAP-TEST or email [email protected]

Prep for an Exam

MCAT Test Prep

LSAT Test Prep

GRE Test Prep

GMAT Test Prep

SAT Test Prep

ACT Test Prep

DAT Test Prep

NCLEX Test Prep

USMLE Test Prep

Courses by Location

NCLEX Locations

GRE Locations

SAT Locations

LSAT Locations

MCAT Locations

GMAT Locations

Useful Links

Kaplan Test Prep Contact Us Partner Solutions Work for Kaplan Terms and Conditions Privacy Policy CA Privacy Policy Trademark Directory

Six Things To Know About AP US History Period 3

Examples logo

AP United States History Score Calculator

The AP United States History Score Calculator by examples.com helps students estimate their exam scores by converting raw scores from multiple-choice and free-response sections into a scaled score, aiding in effective preparation.

How to Calculate AP United States History Score

The AP United States History (APUSH) score is determined by combining raw scores from multiple sections of the exam. Each section contributes to the final scaled score.

1. Multiple-Choice Section:

  • Number of Questions: 55 questions.
  • Scoring: Each correct answer earns one point, with no penalty for incorrect answers.
  • Raw Score: The total number of correct answers forms your raw score for this section, which accounts for 40% of your final score.

2. Short-Answer Section:

  • Number of Questions: 3 questions.
  • Scoring: Each question is scored on a scale from 0 to 3 points.
  • Raw Score: The combined score from all short-answer questions forms the raw score for this section, contributing 20% to your final score.

3. Document-Based Question (DBQ) Section:

  • Number of Questions: 1 question.
  • Scoring: The DBQ is scored on a scale from 0 to 7 points.
  • Raw Score: The score from the DBQ contributes 25% to your final score.

4. Long Essay Section:

  • Number of Questions: 1 question (choose 1 out of 3 prompts).
  • Scoring: The long essay is scored on a scale from 0 to 6 points.
  • Raw Score: The score for this section contributes 15% to your final score.

5. Converting Raw Scores to Scaled Scores:

  • Combination: The raw scores from all sections are combined according to their weightings.
  • Conversion: The combined raw score is converted to a scaled score between 1 and 5 using a conversion table provided by the College Board.
  • Adjustment: This conversion process accounts for exam difficulty, ensuring consistent and fair scoring across different test versions.

The final scaled score, ranging from 1 (lowest) to 5 (highest), represents your overall performance on the AP United States History exam.

IMAGES

  1. ️ Ap world history essay prompts. AP World History Exam: Document. 2019

    long essay question ap world example

  2. AP World History LEQ (Long Essay Question) Introduction Lesson

    long essay question ap world example

  3. AP World unit 9 review questions and answers 100% pass

    long essay question ap world example

  4. AP World History Essay Final

    long essay question ap world example

  5. Ap world essay topics 2016

    long essay question ap world example

  6. AP World History Long Essay Question and Writing Guide

    long essay question ap world example

COMMENTS

  1. AP World History: Modern Sample Long Essay Question

    Step 1: Analyze the Prompt. On the actual exam, you will read three questions and determine which you can answer most confidently. For this sample question, note that you will be evaluating how changes in the spread of ideas impacted societies. The words "changes," "impacted," and "the extent" indicate that this prompt is testing ...

  2. AP World History: Modern Exam Questions

    Download free-response questions from this year's exam and past exams along with scoring guidelines, sample responses from exam takers, and scoring distributions. If you are using assistive technology and need help accessing these PDFs in another format, contact Services for Students with Disabilities at 212-713-8333 or by email at ssd@info ...

  3. How to Approach AP World History: Modern Long Essay Questions

    During Step 1: Analyze the Prompt. Each long essay question begins with a general statement that provides context about the tested time period, and then the second sentence identifies your task, which will always entail developing an evaluative argument. Make sure to read all three prompts carefully. Think of the evidence you could use and the ...

  4. PDF AP World History: Modern

    AP ® World History: Modern Sample Student Responses ... Question 2: Long Essay Question, Economic and Commercial Practices in Afro-Eurasia 6 points General Scoring Notes • Except where otherwise noted, each point of these rubrics is earned independently; for example, a student could earn a point for evidence ...

  5. PDF AP World History

    Question 3 — Long Essay Question (continued) Examples of unacceptable theses (hypothetical): • "The Columbian Exchange transformed the lives of peoples living in the Americas in the period

  6. PDF AP World History

    Question 2 — Long Essay Question (continued) Using Evidence — Examples: Addresses the topic of the question with specific examples of relevant evidence. (1 point) Must address the topic of the question by referring to at least TWO specific examples or pieces of relevant evidence reflecting labor migrations. Essays can earn this point ...

  7. AP World History: Modern Sample DBQ

    Step 2: Plan Your Response. Next, take time to plan your response. Focus on formulating a strong thesis, and check your plan against the six DBQ requirements. See the sample plan that a high-scoring writer might make. Scoring requirements are written in bold for reference; note that the writer includes six of the seven documents and plans to ...

  8. PDF AP World History

    The intent of this question was to assess students' knowledge of continuities and changes in the global balance of political power in the period from 1900 C.E. to the present. This content is part of Key Concepts 6.2 and 6.3 of the AP World History Curriculum Framework (Global Conflicts and Their Consequences and New Conceptualizations of ...

  9. AP World History Long Essay Question Example 1

    Below you can review a sample answer and evaluate what earns this AP World History LEQ example a perfect score. Evaluate the impact of the trans-Saharan trade routes on the exchange of goods, cultures, and ideas in Africa and the Islamic world during the period c. 1200-1750. Analyze the role of trade networks in shaping societies and ...

  10. AP World History How To Write a LEQ Overview

    1815-2001. Writing time on the AP Exam includes both the Document Based Question (DBQ) and the (LEQ), but it is suggested that you spend 40 minutes completing the LEQ. You will need to plan and write your essay in that time. A good breakdown would be 5 min. (planning) + 35 min. (writing) = 40 min. ** Try using a study timer to maximize your ...

  11. PDF AP World History: Modern

    AP ® World History: Modern Sample Student Responses and Scoring Commentary ... Question 4: Long Essay Question, Free-market Ideas and Economic Change 6 points . General Scoring Notes • Except where otherwise noted, each point of these rubrics is earned independently; for example, a student could earn a point for evidence ...

  12. PDF 2022 AP Student Samples and Commentary

    The Document-Based Question (DBQ) asked students to evaluate the extent to which European imperialism had an impact on the economies of Africa and/or Asia. Responses were expected to address the time frame of the 19th through the early 20th centuries and to demonstrate the historical thinking skill of causation.

  13. The Complete Guide to the AP World History Exam

    Sample AP World History Test Questions. ... Long Essay Question Example. For the Long Essay, you'll get three possible prompts to choose from. This question is an example of an Option 2 prompt with a focus on the years 1450-1900—in this case, the 19th century. You can earn up to 6 points for your essay.

  14. Long Essay Question (LEQ)

    Breakdown of Essay: The AP U.S. History exam gives students a choice between two long-essay questions. You chose ONE! A thesis statement is required. You will have 35 minutes to answer the one question you select. Makes up 15 % of final exam score. Graded on a 0-6 point scale.

  15. AP World History Exam: Long Essay Question

    The long essay question on the AP World History exam assesses your ability to apply knowledge of history in a complex, analytical manner. In other words, you are expected to treat history and historical questions as a historian would. This process is called historiography—the skills and strategies historians use to analyze and interpret ...

  16. Question Types on the AP World History: Modern Exam

    There are four types of questions on the AP World History: Modern exam, including multiple-choice questions, short-answer questions, and free-response (essay) questions. The free-response questions are composed of a document-based question (DBQ) and a long essay question (LEQ). In the first half of the exam, you will have 55 minutes to complete ...

  17. PDF AP World History: Modern

    AP® 2021 Scoring Guidelines . Question 4: Long Essay Question, Economic Responses to the Great Depression 6 points . General Scoring Notes • Except where otherwise noted, each point of these rubrics is earned independently; for example, a student could earn a point for evidence without earning a point for thesis/claim. • Accuracy:

  18. AP World History: Modern Exam

    Long Essay Question 40 Minutes | 15% of Score. You'll have a choice of three questions; you'll pick one to answer. Each tests the same skills but the questions focus on different historical time periods (either the period from c. 1200-1750, from c. 1450-1900, or from c. 1750-2001).

  19. AP World LEQ 2

    AP World History LEQ Example: Throughout 1200-1450, the Chinese caused significant alterations in the lifestyle of the western hemisphere. While not all innovations were detrimental in this political revision process, tactics such as the use of the civil service exam and invention of gunpowder are undoubtedly memorable occurrences when discussing Chinese history.

  20. PDF AP World History

    Question 2 — Long Essay Question (continued) Examples of unacceptable theses (hypothetical): • "New religions spread across Afro-Eurasia in the period 600 B .

  21. AP World History: Modern Exam

    Rubrics Updated for 2023-24. We've updated the AP World History: Modern document-based question (DBQ) and long essay question (LEQ) rubrics for the 2023-24 school year. This change only affects the DBQ and LEQ scoring, with no change to the course or the exam: the exam format, course framework, and skills assessed on the exam all remain ...

  22. AP World History Notes: Comparison in the AP Histories

    🌍 Exam Skills study guides written by former AP World students to review Exam Skills with detailed explanations and practice questions. All Subjects. Light. Unit 0 - Before 1200 CE. Unit 1 - The Global Tapestry, 1200-1450 ... India is an excellent example. India had a long standing politically organized independence movement.

  23. AP U.S. History Long Essay Example

    Step 2: Plan Your Response. Next, take time to plan your response. Check your plan against the long essay question requirements. See the sample plan that a high-scoring writer might make; scoring requirements are written in bold for reference. Step 3: Action! Write Your Response & Step 4: Proofread.

  24. AP United States History Score Calculator

    3. Document-Based Question (DBQ) Section: Number of Questions: 1 question. Scoring: The DBQ is scored on a scale from 0 to 7 points. Raw Score: The score from the DBQ contributes 25% to your final score. 4. Long Essay Section: Number of Questions: 1 question (choose 1 out of 3 prompts). Scoring: The long essay is scored on a scale from 0 to 6 ...

  25. PDF 2021 AP Exam Administration Sample Student Responses

    Question 3: Long Essay Question, East and South Asian Economic Responses to Imperialism 6 points General Scoring Notes • Except where otherwise noted, each point of these rubrics is earned independently; for example, a student could earn a point for evidence